LSAT and Law School Admissions Forum

Get expert LSAT preparation and law school admissions advice from PowerScore Test Preparation.

 Administrator
PowerScore Staff
  • PowerScore Staff
  • Posts: 8916
  • Joined: Feb 02, 2011
|
#36264
Complete Question Explanation

Parallel Flaw—#%. The correct answer choice is (C)

The stimulus argues that since the average price of groceries will rise next month, the price of two
specifi c goods—butter and eggs—will rise next month. This is a classic error of division where a
characteristic of the whole is presumed to apply to all of its parts. Specifi cally, in this problem the
conclusion is unwarranted because the increase in the average price of groceries could be attributed
to many items other than butter or eggs. In fact, the price of butter and eggs could have decreased
because the overall qualities of a group are not necessarily shared by each part of the group.

Note that the correct answer must contain a fl aw similar to the one in the stimulus. Any answer
choice featuring valid reasoning or a different type of fl aw should be eliminated.

Answer choice (A): This answer is incorrect on two counts. First, the choice projects a past trend
into the future, whereas the argument in the stimulus involves the future but not the past. Second,
this choice does not use a general average and apply it to specifi c items. Instead, a single item is
addressed throughout the answer.

Answer choice (B): This choice states that only two outcomes are possible, and then concludes that a
since one of the options is unlikely to occur that the other option must occur. Although the reasoning
is fl awed, it is not analogous to that found in the stimulus.

Answer choice (C): This is the correct answer choice. The answer addresses a characteristic
(amount of time spent watching television) of a general group (people younger than 20) and indicates
that the average characteristic is increasing. The answer then concludes that the characteristic of
a part (fourth graders) of the general group must also have risen. Similar to the argument in the
stimulus, this argument ignores the fact that an average increase does not mean that every part of the
group has increased.

Answer choice (D): This choice contains valid conditional reasoning. If the price of ice cream
rises whenever the price of sugar rises, and next month the price of sugar is expected to rise, then it
follows that next month the price of ice cream is expected to rise. Conditionally speaking, one of the
premises contains the following relationship:

..... Price of sugar rises :arrow: Price of ice cream rises

Another premise indicates that next month the suffi cient condition is expected to occur, and thus one
can conclude that next month the necessary condition is likely to occur.

Note that although this argument contains a part to whole relationship, there is not the same fl awed
use of an average and its component parts.

Answer choice (E): This choice also contains conditional reasoning. The fi rst sentence contains both
a conditional relationship and the statement that the suffi cient condition will occur over the next
decade:

..... Conditional Relationship: Population 20-30 declines :arrow: Real estate prices fall

..... Suffi cient condition met: Population 20-30 declines over next decade

Consequently, we can conclude that the necessary condition will also occur over that period, and that
real estate prices fall over the next decade.

Note also that there is no part to whole relationship within this answer, so this answer is incorrect on
that count as well.
 eober
  • Posts: 107
  • Joined: Jul 24, 2014
|
#16305
Hi,

I was wondering in parallel reasoning questions shouldn't be cautious about the language as well? In the conclusion is says "can be expected" but answer choice C says "must have risen". I changed my answer after realizing this. Do they have the same meaning or is the wording not crucial to parallel?

Thanks!
 Steve Stein
PowerScore Staff
  • PowerScore Staff
  • Posts: 1153
  • Joined: Apr 11, 2011
|
#16325
Hi,

That's a good question. Wording can certainly be important, but it really depends on the context. "Can be expected" and "must have risen" do not have the exact same meaning--the first refers to the future and the second to the past. In this case, though, they don't have to mean the exact same thing, because the flaw in the author's reasoning is an error of division--in both the stimulus and the correct answer choice, the author has attributed a characteristic of the whole to a part of that group.

I hope that's helpful! Please let me know whether this is clear--thanks!

~Steve
 ahmed95
  • Posts: 5
  • Joined: Aug 22, 2015
|
#19754
Hello,

For the parallel reasoning question, I chose "D" even though the answer choice was "C" and got completely thrown off by it.

Answer choice C uses 'must' which was not used in 'D' nor in the stimulus. Also, I feel that D and the stimulus look towards the future in the conclusion (i.e. expected to rise next month) while C looks towards the recent past.

I am sure there is a proper explanation as to why C was the correct answer but I cannot seem to figure it out. Look forward to your clarification and thank you in advance!
 David Boyle
PowerScore Staff
  • PowerScore Staff
  • Posts: 836
  • Joined: Jun 07, 2013
|
#19759
ahmed95 wrote:Hello,

For the parallel reasoning question, I chose "D" even though the answer choice was "C" and got completely thrown off by it.

Answer choice C uses 'must' which was not used in 'D' nor in the stimulus. Also, I feel that D and the stimulus look towards the future in the conclusion (i.e. expected to rise next month) while C looks towards the recent past.

I am sure there is a proper explanation as to why C was the correct answer but I cannot seem to figure it out. Look forward to your clarification and thank you in advance!
Hello ahmed95,

True, answer C uses "must" instead of the stimulus' "can be expected to". And answer D does use the near future, like the stimulus does, and even the sort-of-magic words "can be expected to". However, on the whole, answer C is the best, since the logic of C follows the stimulus the most closely.
The stimulus has a false conclusion about a part versus a whole, i.e., just because groceries will rise in price, that doesn't mean that the subset of groceries, "butter and eggs", will follow that average trend. Answer C follows this logic by taking 4th graders out of the larger set of "people under 20", and saying the 4th graders watched more TV just because the larger set as a whole did--and answer C makes an unsupported assumption by doing so. Answer D doesn't really follow this train, though, since it talks about sort of the opposite: a subset (sugar) has its price go up, therefore the larger set (ice cream) has its price go up as well.

Hope this helps,
David
 Pragmatism
  • Posts: 68
  • Joined: Jan 11, 2018
|
#43213
So, the very reason Ahmed chose answer choice D, and the reasoning that justifies his negligence of choosing answer choice C was the very same method that led to me to pick answer choice D. I have a question, in regular parallel questions, which could be valid or invalid, and in flawed parallel questions, does logic take precedence over the logical force tied within the stimulus? And if that is the case, then should one consider the logical force within the conclusion and premise, only when there are two answer choices that encompass the similar logical flaw structurally?
 clairehoran
  • Posts: 2
  • Joined: Feb 22, 2016
|
#43656
Hi Pragmatism,

I'm not sure what you mean by "logical force," but you should consider the type/structure of reasoning first, then whether the argument is valid or invalid. Of course, if you have trouble with the structure of a particular problem, then identifying valid or invalid can help you eliminate some of the wrong answers.
 Pragmatism
  • Posts: 68
  • Joined: Jan 11, 2018
|
#43664
Claire,

By "logical force," I am referring to the strength of the conclusion and or premise. Words such as "probably," or "possibly," or even, "should," could possibly impact the answer choices, however, I have seen answer choices that have a stronger logical force than the stimulus, but match its general logic. So, since I have made marginal errors due to isolating two answer choices, one which conforms to the logical structure of the stimulus, but instead of conforming to the logical force of word "may" in the stimulus, the answer choice will state "should," verses another that matches the logical force, but slightly misses the logical structure of the stimulus, how much weight should be put on the logical structure of the argument and the logical force within the argument. Also, at what point should one consider the logical force within the argument?

Thanks
 Adam Tyson
PowerScore Staff
  • PowerScore Staff
  • Posts: 5153
  • Joined: Apr 14, 2011
|
#43696
Interesting question, pragmatism! We teach that the logical force of the stimulus is one of the controlling factors, so an argument about what may happen should never, ever be parallel to one that is about what should happen, nor should an argument about what is certain be parallel to one that is merely likely. But we also teach that the abstract structure of the arguments should also be parallel. The thing is, those two should not be in conflict - the structure includes the force of the language. It's all wrapped up in one package.

If you find yourself looking at two similar structures, and one matches the force of the language while the other does not, go with the one that matches the force.

Now, on a flawed question like this one, your main focus should be on paralleling the flaw, not the overall structure or force. Those will likely still match up, but the issue is really the flaw. In this case, that is a "whole to part" flaw, which we sometimes call an Error of Division. C has it, D doesn't have it, so C is the better answer. Also, D appears to be valid, so it can't parallel a flawed argument! Try diagramming D conditionally, and you'll find that it is what we call a "repeat form" - the sufficient condition is expected to happen, so the necessary condition is likewise expected.

Not sure I addressed the core issue for you here, but if you have an example of another question where one answer matched the force of language but not the structure, and another matched the structure but not the force of language, please share and we'll revisit this!
 Pragmatism
  • Posts: 68
  • Joined: Jan 11, 2018
|
#43709
Adam, I appreciate your response. A perfect example of what I am referring to is on Pretest 45, 1st LR Section, question #11. The conclusion in the stimulus uses "can be expected" as its logical force, whereas the correct answer choice uses "must have risen." Now, while this was a fairly easy question, I wanted to see at what point does logical structure take precedence over the logical force within the argument?

I hope this example showcases what my question is referring to. Thanks again.

Get the most out of your LSAT Prep Plus subscription.

Analyze and track your performance with our Testing and Analytics Package.